What is the solution to Problem B-5 in the 1997 Putnam Competition?

  • MHB
  • Thread starter Ackbach
  • Start date
  • Tags
    2016
In summary, Problem B-5 in the 1997 Putnam Competition was a challenging mathematical problem presented in the annual William Lowell Putnam Mathematical Competition. The solution to this problem is not publicly available, as the Putnam Competition is a prestigious and competitive event. To approach solving this problem, one would need a strong understanding of advanced mathematics and problem-solving techniques. The skills required to solve it include mathematical proficiency, critical thinking, creativity, and patience. The purpose of the Putnam Competition is to promote excellence in mathematics and encourage students to develop their problem-solving skills and mathematical reasoning abilities.
  • #1
Ackbach
Gold Member
MHB
4,155
89
Here is this week's POTW:

-----

Prove that for $n\geq 2$,
\[
\overbrace{2^{2^{\cdots^{2}}}}^{\mbox{$n$ terms}} \equiv
\overbrace{2^{2^{\cdots^{2}}}}^{\mbox{$n-1$ terms}} \quad \pmod{n}.
\]

-----

Remember to read the http://www.mathhelpboards.com/showthread.php?772-Problem-of-the-Week-%28POTW%29-Procedure-and-Guidelines to find out how to http://www.mathhelpboards.com/forms.php?do=form&fid=2!
 
Physics news on Phys.org
  • #2
Re: Problem Of The Week # 236 - Oct 06, 2016

This was Problem B-5 in the 1997 William Lowell Putnam Mathematical Competition.

No one answered this week's POTW. The solution, attributed to Kiran Kedlaya and his associates, follows:

Define the sequence $x_1 = 2$, $x_n = 2^{x_{n-1}}$ for $n > 1$. It suffices to show that for every $n$, $x_m \equiv x_{m+1} \equiv \cdots \pmod n$ for some $m < n.$ We do this by induction on $n$, with $n=2$ being obvious.

Write $n = 2^a b$, where $b$ is odd. It suffices to show that $x_m \equiv \cdots$ modulo $2^a$ and modulo $b$, for some $m < n$. For the former, we only need $x_{n-1} \geq a$, but clearly $x_{n-1} \geq n$ by induction on $n$. For the latter, note that $x_m \equiv x_{m+1} \equiv \cdots\pmod b$ as long as $x_{m-1} \equiv x_m \equiv \cdots \pmod{\phi(b)}$, where $\phi(n)$ is the Euler totient function. By hypothesis, this occurs for some $m < \phi(b) + 1 \leq n$.
 

Related to What is the solution to Problem B-5 in the 1997 Putnam Competition?

What is Problem B-5 in the 1997 Putnam Competition?

Problem B-5 in the 1997 Putnam Competition was a mathematical problem that appeared in the annual William Lowell Putnam Mathematical Competition. It is a challenging problem that requires creative and advanced problem-solving skills.

What is the solution to Problem B-5 in the 1997 Putnam Competition?

The solution to Problem B-5 in the 1997 Putnam Competition is not publicly available. The Putnam Competition is a prestigious and highly competitive event, and solutions to the problems are not released to the public.

How do you approach solving Problem B-5 in the 1997 Putnam Competition?

To solve Problem B-5 in the 1997 Putnam Competition, one would need a strong understanding of advanced mathematics and problem-solving techniques. It is important to carefully read and understand the problem, break it down into smaller parts, and use logical reasoning to come up with a solution.

What skills are required to solve Problem B-5 in the 1997 Putnam Competition?

Solving Problem B-5 in the 1997 Putnam Competition requires a high level of mathematical proficiency, critical thinking skills, and the ability to think creatively and outside the box. It also requires patience, persistence, and the ability to work through complex problems.

What is the purpose of the Putnam Competition?

The Putnam Competition is an annual mathematics competition for undergraduate students in the United States and Canada. Its purpose is to promote excellence in mathematics and to encourage students to develop their problem-solving skills and mathematical reasoning abilities.

Similar threads

Replies
1
Views
1K
Replies
1
Views
1K
Replies
1
Views
1K
  • Math POTW for University Students
Replies
1
Views
1K
  • Math POTW for University Students
Replies
1
Views
1K
  • Math POTW for University Students
Replies
1
Views
1K
  • Math POTW for University Students
Replies
1
Views
1K
Replies
1
Views
1K
  • Math POTW for University Students
Replies
1
Views
1K
Back
Top